Difference between revisions of "1992 AHSME Problems/Problem 17"
Johnhankock (talk | contribs) (→Solution) |
Mathpro12345 (talk | contribs) (→Solution 2) |
||
(12 intermediate revisions by 4 users not shown) | |||
Line 9: | Line 9: | ||
== Solution == | == Solution == | ||
+ | === Solution 1=== | ||
− | + | We can determine if our number is divisible by <math>3</math> or <math>9</math> by summing the digits. Looking at the one's place, we can start out with <math>0, 1, 2, 3, 4, 5, 6, 7, 8, 9</math> and continue cycling though the numbers from <math>0</math> through <math>9</math>. For each one of these cycles, we add <math>0 + 1 + ... + 9 = 45</math>. This is divisible by <math>9</math>, thus we can ignore the sum. However, this excludes <math>19</math>, <math>90</math>, <math>91</math> and <math>92</math>. These remaining units digits sum up to <math>9 + 1 + 2 = 12</math>, which means our units sum is <math>3 \pmod 9</math>. As for the tens digits, for <math>2, 3, 4, \cdots , 8</math> we have <math>10</math> sets of those: <cmath>\frac{8 \cdot 9}{2} - 1 = 35,</cmath> which is congruent to <math>8 \pmod 9</math>. We again have <math>19, 90, 91</math> and <math>92</math>, so we must add <cmath>1 + 9 \cdot 3 = 28</cmath> to our total. <math>28</math> is congruent to <math>1 \pmod 9</math>. Thus our sum is congruent to <math>3 \pmod 9</math>, and <math>k = 1 | |
+ | \implies \boxed{B}</math>. | ||
+ | |||
+ | ==Solution 2== | ||
+ | |||
+ | As our first trial with 3, We can say that <math>N\equiv 1+9+2+0+2+1+2+2+...+9+1+9+2\pmod{3}</math>. Since if the sum of the digits of a number is divisible by 3, then the number is also divisible by 3, we reverse that logic to say that <math>N\equiv 19+20+21+22+...+91+92\pmod{3}</math>, and adding that up using the rainbow strategy, we get <math>N\equiv (111\times37)\pmod{3}</math>. We can see that since 3 divides 111, the original number is divisible by 3. But, since 111 only has one 3 and 37 has no 3's, it is not divisble by 9. Thus, the answer is <math>\boxed{B}</math> | ||
+ | |||
+ | ~mathpro12345 | ||
+ | |||
+ | papa I wrote this second one | ||
== See also == | == See also == |
Latest revision as of 12:19, 4 January 2021
Problem
The 2-digit integers from 19 to 92 are written consecutively to form the integer . Suppose that is the highest power of 3 that is a factor of . What is ?
Solution
Solution 1
We can determine if our number is divisible by or by summing the digits. Looking at the one's place, we can start out with and continue cycling though the numbers from through . For each one of these cycles, we add . This is divisible by , thus we can ignore the sum. However, this excludes , , and . These remaining units digits sum up to , which means our units sum is . As for the tens digits, for we have sets of those: which is congruent to . We again have and , so we must add to our total. is congruent to . Thus our sum is congruent to , and .
Solution 2
As our first trial with 3, We can say that . Since if the sum of the digits of a number is divisible by 3, then the number is also divisible by 3, we reverse that logic to say that , and adding that up using the rainbow strategy, we get . We can see that since 3 divides 111, the original number is divisible by 3. But, since 111 only has one 3 and 37 has no 3's, it is not divisble by 9. Thus, the answer is
~mathpro12345
papa I wrote this second one
See also
1992 AHSME (Problems • Answer Key • Resources) | ||
Preceded by Problem 16 |
Followed by Problem 18 | |
1 • 2 • 3 • 4 • 5 • 6 • 7 • 8 • 9 • 10 • 11 • 12 • 13 • 14 • 15 • 16 • 17 • 18 • 19 • 20 • 21 • 22 • 23 • 24 • 25 • 26 • 27 • 28 • 29 • 30 | ||
All AHSME Problems and Solutions |
The problems on this page are copyrighted by the Mathematical Association of America's American Mathematics Competitions.